PT36.S3.Q26 - before 1986 physicists

AnthonyScaliaAnthonyScalia Alum Member
edited July 2017 in Logical Reasoning 330 karma

My question here stems from the situation presented in PT36.S3.Q26 (https://7sage.com/lsat_explanations/lsat-36-section-3-question-26/)

It seems that the explanation for the right answer is that by falsifying a factor that would harm an argument if otherwise true, you are therefor strengthening it. This seems odd to me, as establishing a lack of past dissent doesn't equal some degree of current support in my opinion. It's neutral if anything.

But anyway, if we accept the aforementioned logic as fact for LSAT purposes, here are my

Does this mean that scratching off a potential weakening factor always counts, and will always be the answer for a "Most Strengthen" question?

And if so, does it happen in the reverse? Does scratching off a potential strengthener always qualify as the answer which most weakens?

Would we ever have to choose between an answer that merely hurts a weakening factor (which I see as being closer to neutral) vs. deliberately supports a strengthening factor?

Thanks ^_^

Comments

  • Rigid DesignatorRigid Designator Alum Member
    1091 karma

    I don't think this will always be the structure of correct answers for Strengthen questions, and I don't think the overall strategy works in reverse. But eliminating a potential weakness with the argument will always strengthen an argument.

    Look at it this way... all arguments make certain assumptions. Say we've got argument A that makes assumptions X, Y, Z. Now if it turned out that assumption Z was an incorrect assumption our argument would be in trouble. That's a potential weakness of argument A - it's assumption Z might be wrong. Now suppose someone comes along and says 'guys, don't worry, I've run tests which show Z is a perfectly reasonable assumption'. Great, now we have eliminated a potential problem with argument A, namely that its assumption Z might've been wrong. Is argument A now better off? I think most would agree that it is and that we've made the argument better, or have strengthened it. At the very least I'd say it's not a neutral result. The argument has one less potential weakness, and that's not a neutral result.

Sign In or Register to comment.